LSAT and Law School Admissions Forum

Get expert LSAT preparation and law school admissions advice from PowerScore Test Preparation.

 Jeremy Press
PowerScore Staff
  • PowerScore Staff
  • Posts: 1000
  • Joined: Jun 12, 2017
|
#74458
Hi Katya,

You can definitely connect two rules through the contrapositive of one of the rules and the non-contrapositive form of the other rule. This works logically, because the contrapositive of any rule is, in essence, the logical equivalent of its non-contrapositive. So you can make connections between two non-contrapositives, two contrapositives, or a contrapositive and a non-contrapositive. I hope this helps!

Jeremy
 Katya W
  • Posts: 42
  • Joined: Dec 03, 2019
|
#74634
Jeremy Press wrote:Hi Katya,

You can definitely connect two rules through the contrapositive of one of the rules and the non-contrapositive form of the other rule. This works logically, because the contrapositive of any rule is, in essence, the logical equivalent of its non-contrapositive. So you can make connections between two non-contrapositives, two contrapositives, or a contrapositive and a non-contrapositive. I hope this helps!

Jeremy
Wow, that is interesting. Thank you for illuminating that fact for me!
 leslie7
  • Posts: 73
  • Joined: Oct 06, 2020
|
#83134
Hello ,

I got this question correct based off of inferences but I didn't feel confident or 100% sure that the others were wrong. So I went back and still diagrammed each one to be sure that the answer I selected was correct.

Would it have been possible to feel 100% confident about the correct answer in this specific question? (maybe I am missing something)

And on the LSAT (real test day) I guess if I am not 100% confident is the best thing to do, to just select it without considering the others and moving on? (since diagramming each one took a lot of time)

Unless there is an obvious way that I should have known that the answer was 100% correct that maybe I am blind to?

Let me know ty ! :D
 Adam Tyson
PowerScore Staff
  • PowerScore Staff
  • Posts: 5153
  • Joined: Apr 14, 2011
|
#84089
The first thing to do with a local question like this, leslie7, is to diagram the local restriction. Draw out the linear base of 6 slots and place L last. Now, what rule does that trigger? L is not before O, so Z cannot be first - the contrapositive of the second rule. Perhaps draw a not-law for Z under the first slot. Having made that deduction (and you should be 100% confident of that, because that's how conditional rules and contrapositives work, as 100% guarantees), you should check the answers and see if you have proven that one of them must be true. When you see that you have proven answer A, you should just select it and move to the next question!

Trust the conditions, and the contrapositives. Trust your inferences, because if you don't then there is no point making them in the first place! Mostly, trust yourself. because to do otherwise is to waste time on second guessing and doing more work than the questions require, losing the chance to get further into the section selecting more and more right answers.
 g_lawyered
  • Posts: 211
  • Joined: Sep 14, 2020
|
#89063
Hi P.S.,
I, too, had a hard time making inferences before going to the questions. Because I was under the impression that I shouldn't take time to infer anything from conditional statements because they are just "conditional". Meaning that if the questions didn't ask or trigger the conditional statements, then I would've wasted that time making inferences instead of getting into the questions.
Because of this I took a long time to answer Question #3 since I was sketching out different diagrams. I read the explanation and understand that answer choice A is correct because it tests rule 2 contrapositive (and if I would've made the inference in the explanation I didn't have to test out answer choices).
But my question is: Why is answer choice D not a MBT? I chose this answer because I drew it out as (Z being 3 as I made the inference that Z couldn't be 1st and M couldn't be 3rd):
K/T, M, Z, T/K, O, L

Can someone explain why this isn't correct answer? Is it a could be true answer and not a must be true answer? :-?
Thanks in advance
 Adam Tyson
PowerScore Staff
  • PowerScore Staff
  • Posts: 5153
  • Joined: Apr 14, 2011
|
#89784
Answer D is indeed a Could Be True but not a Must Be True answer, GGIBA003@FIU.EDU, because Z could be 2nd, 3rd, 4th, or 5th in this local scenario. M actually COULD be 3rd, since the necessary condition for M going 3rd (L is last) has been satisfied. For example, this solution would work:

TOMKZL

But so could this one:

KMTZOL

Conditional rules can and often do lead to inferences. For example, imagine that this game also had this rule:

"If K is before M, Z is first."

From this rule we could infer that K will never be first. If it was first, it would be before M, but that would require Z to be first, setting up an impossible conflict. It doesn't matter that the rule is conditional; that inference is an absolute requirement whether the rule is triggered (the sufficient condition happens) or not! Start looking for implications like that from conditional rules and you'll find that they can be much more "active" than they may at first appear.
 g_lawyered
  • Posts: 211
  • Joined: Sep 14, 2020
|
#89832
The additional rule you added to this game was helpful for me to see how inferences can be made from conditional rules. Thank you Adam! Since this game doesn't have such a rule can you tell me a specific game where an inference of that kind comes from the rules? That way I can practice what you advise. In the meantime, I will try to incorporate your advice into my LG practice.
Thanks in advance!
 Robert Carroll
PowerScore Staff
  • PowerScore Staff
  • Posts: 1783
  • Joined: Dec 06, 2013
|
#90130
GGIBA,

There actually is such a real case! In the October 2012 test (Preptest 67), in the second game, the last rule ("Powell lectures first...") gives a situation where Y cannot be first, because of the last rule. Check it out: viewtopic.php?f=105&t=5457

specifically in the setup first post:

The conditional relationship between P, Y, and V leads to a complex, but critical Not Law: if Y lectured first, Y would be lecturing before any of the other six variables, including V. But, due to the bi-directional nature of the double-arrow, this Y :longline: V sequence would require P to lecture first, which is a logical impossibility since only one professor can lecture at any given time. To avoid this paradox, Y cannot lecture first. Consequently, only P, T, or V can lecture first.
That's a little different because it's a biconditional, but the second half of that biconditional is a situation just like Adam's conditional above!

Robert Carroll
User avatar
 hassanahsmith93
  • Posts: 3
  • Joined: Mar 26, 2022
|
#96128
If L is 6 why is M3 not the answer. I understand that Z wouldn't be one, but I don't understand why we would not select M3 as the answer choice if both are right.
User avatar
 Stephanie Oswalt
PowerScore Staff
  • PowerScore Staff
  • Posts: 804
  • Joined: Jan 11, 2016
|
#96162
hassanahsmith93 wrote: Mon Jul 11, 2022 4:14 pm If L is 6 why is M3 not the answer. I understand that Z wouldn't be one, but I don't understand why we would not select M3 as the answer choice if both are right.
Hi Hassanah,

Thanks for the post! I believe you are asking about Question #3, so I have moved your post to the thread discussing this topic. Please review the explanation and discussion starting on page 1 here: viewtopic.php?f=239&t=15713 and let us know if that helps, or if you still have further questions!

Thank you!

Get the most out of your LSAT Prep Plus subscription.

Analyze and track your performance with our Testing and Analytics Package.